Another beauty hidden in a simple triangle (2)

The name of the pictureThe name of the pictureThe name of the pictureClash Royale CLAN TAG#URR8PPP











up vote
17
down vote

favorite
2













Let the inscribed circle of triangle $ABC$ touch its sides $BC, CA, AB$ at $F, G, E$. Denote the center of this circle by $D$, and the midpoint of $BC$ by $M$. Prove that lines $AM$, $EG$ and $DF$ concur.




enter image description here



My idea was to consider the intersection of lines $EG$ and $FD$ first, then to draw a line through intersection point $H$ parallel to $BC$. That line will intersect triangle at some points $I$ and $J$ (not shown). If I could prove that $IH=JH$, that would automatically prove that $AM$ has to pass through point $H$ as well. However, this day was so hot here in my city that I could not think more about this problem. I can only hope that it's more pleasant elsewhere :)







share|cite|improve this question


















  • 1




    The straightforward way is to compute the ratios $sin measuredangle BAH / sin measuredangle HAC$ and $sin measuredangle BAM / sin measuredangle MAC$ (both can be shown to equal $b / c$ using the sine law in triangles $EAH$, $HAG$, $EFH$, $HFG$, $ABM$ and $AMC$), and argue that the equality of these ratios yields that the lines $AH$ and $AM$ are identical. There is likely to be a nicer proof.
    – darij grinberg
    Aug 25 at 20:40







  • 1




    @darijgrinberg Thanks, but the problem comes from the geometry book that does not cover trigonometry at all. There has to be some other solution.
    – Oldboy
    Aug 25 at 20:43






  • 1




    The trilinear (or barycentric) coordinates of $E,F,G,D,M$ are straightforward to compute, so they are the equations of $EG,AM,DF$. The given concurrency boils down to checking that a simple determinant is zero.
    – Jack D'Aurizio♦
    Aug 25 at 20:44










  • @Oldboy Um, I'm so sorry for you not to adopt any of my proofs...
    – mengdie1982
    Aug 25 at 22:56















up vote
17
down vote

favorite
2













Let the inscribed circle of triangle $ABC$ touch its sides $BC, CA, AB$ at $F, G, E$. Denote the center of this circle by $D$, and the midpoint of $BC$ by $M$. Prove that lines $AM$, $EG$ and $DF$ concur.




enter image description here



My idea was to consider the intersection of lines $EG$ and $FD$ first, then to draw a line through intersection point $H$ parallel to $BC$. That line will intersect triangle at some points $I$ and $J$ (not shown). If I could prove that $IH=JH$, that would automatically prove that $AM$ has to pass through point $H$ as well. However, this day was so hot here in my city that I could not think more about this problem. I can only hope that it's more pleasant elsewhere :)







share|cite|improve this question


















  • 1




    The straightforward way is to compute the ratios $sin measuredangle BAH / sin measuredangle HAC$ and $sin measuredangle BAM / sin measuredangle MAC$ (both can be shown to equal $b / c$ using the sine law in triangles $EAH$, $HAG$, $EFH$, $HFG$, $ABM$ and $AMC$), and argue that the equality of these ratios yields that the lines $AH$ and $AM$ are identical. There is likely to be a nicer proof.
    – darij grinberg
    Aug 25 at 20:40







  • 1




    @darijgrinberg Thanks, but the problem comes from the geometry book that does not cover trigonometry at all. There has to be some other solution.
    – Oldboy
    Aug 25 at 20:43






  • 1




    The trilinear (or barycentric) coordinates of $E,F,G,D,M$ are straightforward to compute, so they are the equations of $EG,AM,DF$. The given concurrency boils down to checking that a simple determinant is zero.
    – Jack D'Aurizio♦
    Aug 25 at 20:44










  • @Oldboy Um, I'm so sorry for you not to adopt any of my proofs...
    – mengdie1982
    Aug 25 at 22:56













up vote
17
down vote

favorite
2









up vote
17
down vote

favorite
2






2






Let the inscribed circle of triangle $ABC$ touch its sides $BC, CA, AB$ at $F, G, E$. Denote the center of this circle by $D$, and the midpoint of $BC$ by $M$. Prove that lines $AM$, $EG$ and $DF$ concur.




enter image description here



My idea was to consider the intersection of lines $EG$ and $FD$ first, then to draw a line through intersection point $H$ parallel to $BC$. That line will intersect triangle at some points $I$ and $J$ (not shown). If I could prove that $IH=JH$, that would automatically prove that $AM$ has to pass through point $H$ as well. However, this day was so hot here in my city that I could not think more about this problem. I can only hope that it's more pleasant elsewhere :)







share|cite|improve this question















Let the inscribed circle of triangle $ABC$ touch its sides $BC, CA, AB$ at $F, G, E$. Denote the center of this circle by $D$, and the midpoint of $BC$ by $M$. Prove that lines $AM$, $EG$ and $DF$ concur.




enter image description here



My idea was to consider the intersection of lines $EG$ and $FD$ first, then to draw a line through intersection point $H$ parallel to $BC$. That line will intersect triangle at some points $I$ and $J$ (not shown). If I could prove that $IH=JH$, that would automatically prove that $AM$ has to pass through point $H$ as well. However, this day was so hot here in my city that I could not think more about this problem. I can only hope that it's more pleasant elsewhere :)









share|cite|improve this question













share|cite|improve this question




share|cite|improve this question








edited Aug 25 at 20:39









darij grinberg

9,32632960




9,32632960










asked Aug 25 at 20:20









Oldboy

2,9221318




2,9221318







  • 1




    The straightforward way is to compute the ratios $sin measuredangle BAH / sin measuredangle HAC$ and $sin measuredangle BAM / sin measuredangle MAC$ (both can be shown to equal $b / c$ using the sine law in triangles $EAH$, $HAG$, $EFH$, $HFG$, $ABM$ and $AMC$), and argue that the equality of these ratios yields that the lines $AH$ and $AM$ are identical. There is likely to be a nicer proof.
    – darij grinberg
    Aug 25 at 20:40







  • 1




    @darijgrinberg Thanks, but the problem comes from the geometry book that does not cover trigonometry at all. There has to be some other solution.
    – Oldboy
    Aug 25 at 20:43






  • 1




    The trilinear (or barycentric) coordinates of $E,F,G,D,M$ are straightforward to compute, so they are the equations of $EG,AM,DF$. The given concurrency boils down to checking that a simple determinant is zero.
    – Jack D'Aurizio♦
    Aug 25 at 20:44










  • @Oldboy Um, I'm so sorry for you not to adopt any of my proofs...
    – mengdie1982
    Aug 25 at 22:56













  • 1




    The straightforward way is to compute the ratios $sin measuredangle BAH / sin measuredangle HAC$ and $sin measuredangle BAM / sin measuredangle MAC$ (both can be shown to equal $b / c$ using the sine law in triangles $EAH$, $HAG$, $EFH$, $HFG$, $ABM$ and $AMC$), and argue that the equality of these ratios yields that the lines $AH$ and $AM$ are identical. There is likely to be a nicer proof.
    – darij grinberg
    Aug 25 at 20:40







  • 1




    @darijgrinberg Thanks, but the problem comes from the geometry book that does not cover trigonometry at all. There has to be some other solution.
    – Oldboy
    Aug 25 at 20:43






  • 1




    The trilinear (or barycentric) coordinates of $E,F,G,D,M$ are straightforward to compute, so they are the equations of $EG,AM,DF$. The given concurrency boils down to checking that a simple determinant is zero.
    – Jack D'Aurizio♦
    Aug 25 at 20:44










  • @Oldboy Um, I'm so sorry for you not to adopt any of my proofs...
    – mengdie1982
    Aug 25 at 22:56








1




1




The straightforward way is to compute the ratios $sin measuredangle BAH / sin measuredangle HAC$ and $sin measuredangle BAM / sin measuredangle MAC$ (both can be shown to equal $b / c$ using the sine law in triangles $EAH$, $HAG$, $EFH$, $HFG$, $ABM$ and $AMC$), and argue that the equality of these ratios yields that the lines $AH$ and $AM$ are identical. There is likely to be a nicer proof.
– darij grinberg
Aug 25 at 20:40





The straightforward way is to compute the ratios $sin measuredangle BAH / sin measuredangle HAC$ and $sin measuredangle BAM / sin measuredangle MAC$ (both can be shown to equal $b / c$ using the sine law in triangles $EAH$, $HAG$, $EFH$, $HFG$, $ABM$ and $AMC$), and argue that the equality of these ratios yields that the lines $AH$ and $AM$ are identical. There is likely to be a nicer proof.
– darij grinberg
Aug 25 at 20:40





1




1




@darijgrinberg Thanks, but the problem comes from the geometry book that does not cover trigonometry at all. There has to be some other solution.
– Oldboy
Aug 25 at 20:43




@darijgrinberg Thanks, but the problem comes from the geometry book that does not cover trigonometry at all. There has to be some other solution.
– Oldboy
Aug 25 at 20:43




1




1




The trilinear (or barycentric) coordinates of $E,F,G,D,M$ are straightforward to compute, so they are the equations of $EG,AM,DF$. The given concurrency boils down to checking that a simple determinant is zero.
– Jack D'Aurizio♦
Aug 25 at 20:44




The trilinear (or barycentric) coordinates of $E,F,G,D,M$ are straightforward to compute, so they are the equations of $EG,AM,DF$. The given concurrency boils down to checking that a simple determinant is zero.
– Jack D'Aurizio♦
Aug 25 at 20:44












@Oldboy Um, I'm so sorry for you not to adopt any of my proofs...
– mengdie1982
Aug 25 at 22:56





@Oldboy Um, I'm so sorry for you not to adopt any of my proofs...
– mengdie1982
Aug 25 at 22:56











4 Answers
4






active

oldest

votes

















up vote
3
down vote



accepted










You had the right idea. From $H$ draw a line parallel to $BC$ and call its intersections with $AB$ and $AC$ in order $I$ and $J$. Note that the quadrilaterals $IHDF$ and $HGJD$ are inscribed since $angle DGJ=angle DHJ$ and $angle IHD=angle IFD$ are all right angles. It follows that $angle GDJ=angle GHJ=angle IHF=angle IDF$. So the two triangles $triangle DFI$ and $triangle DGJ$ are congruent for having a leg and a non-right angle congruent. It follows that $DJ$ and $DI$ are congruent and so $HJ$ is congruent to $HI$ and so the line through $A$ and $H$ must go through $M$.






share|cite|improve this answer






















  • I think that you have some typos in your proof but it looks good. The simplest one so far.
    – Oldboy
    Aug 25 at 22:17










  • @Oldboy, yes I fixed some typos. Thanks.
    – Marco
    Aug 25 at 22:22

















up vote
3
down vote













Proof



Let $AM$ intersect $EG$ at $H$. We are to prove $HF$ is the line of the diameter.



Draw a line $l$ passing through $A$ and parallel to $BC$. Let $EG$ intersect $l$ at $X$.
Thus, we may readily see that $(AB,AC|AM,AX)=-1,$ which shows that the polar line of $H$ with respect to the incircle is $AX$. Hence, $DH perp AX ||BC$, then $F,D,H$ are colinear. The proof is completed so far.



enter image description here






share|cite|improve this answer






















  • Please don't use JPG for non-photographic images.
    – Andreas Rejbrand
    Aug 26 at 9:41

















up vote
2
down vote













Another Proof



Denote the the antipodal point of $F$ as $J$. Draw a line $l$ passing through $J$, parallel to $BC$, and intersecting $AB, AC$ at $K, L$ respectively.



Notice that $BCLK$ is a tangential quadrilateral. By Newton's Theorem(Page 156-157) ,which is a degenerated form of Brianchon's theorem, we may obtain $BL,CK,FJ$ are concurrent, namely at $H$. But $KL||BC$, hence $AH$ bisects $BC$ and $KL$. We are done.



enter image description here






share|cite|improve this answer





























    up vote
    1
    down vote













    $IH = JH$ because $M$ is the center point of BC and triangle $AIJ$ is similar with respect to the intersection of its parallel base with line $AM$.
    However, you have to show that these two sides are the same due to symmetry about the line $FD$, so the extension of $FD$ is shown to meet the intersection at $H$.



    The following diagram achieves this by adding a symmetrical construction about $FD$ with $X = Y$.



    enter image description here






    share|cite|improve this answer




















      Your Answer




      StackExchange.ifUsing("editor", function ()
      return StackExchange.using("mathjaxEditing", function ()
      StackExchange.MarkdownEditor.creationCallbacks.add(function (editor, postfix)
      StackExchange.mathjaxEditing.prepareWmdForMathJax(editor, postfix, [["$", "$"], ["\\(","\\)"]]);
      );
      );
      , "mathjax-editing");

      StackExchange.ready(function()
      var channelOptions =
      tags: "".split(" "),
      id: "69"
      ;
      initTagRenderer("".split(" "), "".split(" "), channelOptions);

      StackExchange.using("externalEditor", function()
      // Have to fire editor after snippets, if snippets enabled
      if (StackExchange.settings.snippets.snippetsEnabled)
      StackExchange.using("snippets", function()
      createEditor();
      );

      else
      createEditor();

      );

      function createEditor()
      StackExchange.prepareEditor(
      heartbeatType: 'answer',
      convertImagesToLinks: true,
      noModals: false,
      showLowRepImageUploadWarning: true,
      reputationToPostImages: 10,
      bindNavPrevention: true,
      postfix: "",
      noCode: true, onDemand: true,
      discardSelector: ".discard-answer"
      ,immediatelyShowMarkdownHelp:true
      );



      );













       

      draft saved


      draft discarded


















      StackExchange.ready(
      function ()
      StackExchange.openid.initPostLogin('.new-post-login', 'https%3a%2f%2fmath.stackexchange.com%2fquestions%2f2894467%2fanother-beauty-hidden-in-a-simple-triangle-2%23new-answer', 'question_page');

      );

      Post as a guest






























      4 Answers
      4






      active

      oldest

      votes








      4 Answers
      4






      active

      oldest

      votes









      active

      oldest

      votes






      active

      oldest

      votes








      up vote
      3
      down vote



      accepted










      You had the right idea. From $H$ draw a line parallel to $BC$ and call its intersections with $AB$ and $AC$ in order $I$ and $J$. Note that the quadrilaterals $IHDF$ and $HGJD$ are inscribed since $angle DGJ=angle DHJ$ and $angle IHD=angle IFD$ are all right angles. It follows that $angle GDJ=angle GHJ=angle IHF=angle IDF$. So the two triangles $triangle DFI$ and $triangle DGJ$ are congruent for having a leg and a non-right angle congruent. It follows that $DJ$ and $DI$ are congruent and so $HJ$ is congruent to $HI$ and so the line through $A$ and $H$ must go through $M$.






      share|cite|improve this answer






















      • I think that you have some typos in your proof but it looks good. The simplest one so far.
        – Oldboy
        Aug 25 at 22:17










      • @Oldboy, yes I fixed some typos. Thanks.
        – Marco
        Aug 25 at 22:22














      up vote
      3
      down vote



      accepted










      You had the right idea. From $H$ draw a line parallel to $BC$ and call its intersections with $AB$ and $AC$ in order $I$ and $J$. Note that the quadrilaterals $IHDF$ and $HGJD$ are inscribed since $angle DGJ=angle DHJ$ and $angle IHD=angle IFD$ are all right angles. It follows that $angle GDJ=angle GHJ=angle IHF=angle IDF$. So the two triangles $triangle DFI$ and $triangle DGJ$ are congruent for having a leg and a non-right angle congruent. It follows that $DJ$ and $DI$ are congruent and so $HJ$ is congruent to $HI$ and so the line through $A$ and $H$ must go through $M$.






      share|cite|improve this answer






















      • I think that you have some typos in your proof but it looks good. The simplest one so far.
        – Oldboy
        Aug 25 at 22:17










      • @Oldboy, yes I fixed some typos. Thanks.
        – Marco
        Aug 25 at 22:22












      up vote
      3
      down vote



      accepted







      up vote
      3
      down vote



      accepted






      You had the right idea. From $H$ draw a line parallel to $BC$ and call its intersections with $AB$ and $AC$ in order $I$ and $J$. Note that the quadrilaterals $IHDF$ and $HGJD$ are inscribed since $angle DGJ=angle DHJ$ and $angle IHD=angle IFD$ are all right angles. It follows that $angle GDJ=angle GHJ=angle IHF=angle IDF$. So the two triangles $triangle DFI$ and $triangle DGJ$ are congruent for having a leg and a non-right angle congruent. It follows that $DJ$ and $DI$ are congruent and so $HJ$ is congruent to $HI$ and so the line through $A$ and $H$ must go through $M$.






      share|cite|improve this answer














      You had the right idea. From $H$ draw a line parallel to $BC$ and call its intersections with $AB$ and $AC$ in order $I$ and $J$. Note that the quadrilaterals $IHDF$ and $HGJD$ are inscribed since $angle DGJ=angle DHJ$ and $angle IHD=angle IFD$ are all right angles. It follows that $angle GDJ=angle GHJ=angle IHF=angle IDF$. So the two triangles $triangle DFI$ and $triangle DGJ$ are congruent for having a leg and a non-right angle congruent. It follows that $DJ$ and $DI$ are congruent and so $HJ$ is congruent to $HI$ and so the line through $A$ and $H$ must go through $M$.







      share|cite|improve this answer














      share|cite|improve this answer



      share|cite|improve this answer








      edited Aug 25 at 22:21

























      answered Aug 25 at 21:55









      Marco

      1,55917




      1,55917











      • I think that you have some typos in your proof but it looks good. The simplest one so far.
        – Oldboy
        Aug 25 at 22:17










      • @Oldboy, yes I fixed some typos. Thanks.
        – Marco
        Aug 25 at 22:22
















      • I think that you have some typos in your proof but it looks good. The simplest one so far.
        – Oldboy
        Aug 25 at 22:17










      • @Oldboy, yes I fixed some typos. Thanks.
        – Marco
        Aug 25 at 22:22















      I think that you have some typos in your proof but it looks good. The simplest one so far.
      – Oldboy
      Aug 25 at 22:17




      I think that you have some typos in your proof but it looks good. The simplest one so far.
      – Oldboy
      Aug 25 at 22:17












      @Oldboy, yes I fixed some typos. Thanks.
      – Marco
      Aug 25 at 22:22




      @Oldboy, yes I fixed some typos. Thanks.
      – Marco
      Aug 25 at 22:22










      up vote
      3
      down vote













      Proof



      Let $AM$ intersect $EG$ at $H$. We are to prove $HF$ is the line of the diameter.



      Draw a line $l$ passing through $A$ and parallel to $BC$. Let $EG$ intersect $l$ at $X$.
      Thus, we may readily see that $(AB,AC|AM,AX)=-1,$ which shows that the polar line of $H$ with respect to the incircle is $AX$. Hence, $DH perp AX ||BC$, then $F,D,H$ are colinear. The proof is completed so far.



      enter image description here






      share|cite|improve this answer






















      • Please don't use JPG for non-photographic images.
        – Andreas Rejbrand
        Aug 26 at 9:41














      up vote
      3
      down vote













      Proof



      Let $AM$ intersect $EG$ at $H$. We are to prove $HF$ is the line of the diameter.



      Draw a line $l$ passing through $A$ and parallel to $BC$. Let $EG$ intersect $l$ at $X$.
      Thus, we may readily see that $(AB,AC|AM,AX)=-1,$ which shows that the polar line of $H$ with respect to the incircle is $AX$. Hence, $DH perp AX ||BC$, then $F,D,H$ are colinear. The proof is completed so far.



      enter image description here






      share|cite|improve this answer






















      • Please don't use JPG for non-photographic images.
        – Andreas Rejbrand
        Aug 26 at 9:41












      up vote
      3
      down vote










      up vote
      3
      down vote









      Proof



      Let $AM$ intersect $EG$ at $H$. We are to prove $HF$ is the line of the diameter.



      Draw a line $l$ passing through $A$ and parallel to $BC$. Let $EG$ intersect $l$ at $X$.
      Thus, we may readily see that $(AB,AC|AM,AX)=-1,$ which shows that the polar line of $H$ with respect to the incircle is $AX$. Hence, $DH perp AX ||BC$, then $F,D,H$ are colinear. The proof is completed so far.



      enter image description here






      share|cite|improve this answer














      Proof



      Let $AM$ intersect $EG$ at $H$. We are to prove $HF$ is the line of the diameter.



      Draw a line $l$ passing through $A$ and parallel to $BC$. Let $EG$ intersect $l$ at $X$.
      Thus, we may readily see that $(AB,AC|AM,AX)=-1,$ which shows that the polar line of $H$ with respect to the incircle is $AX$. Hence, $DH perp AX ||BC$, then $F,D,H$ are colinear. The proof is completed so far.



      enter image description here







      share|cite|improve this answer














      share|cite|improve this answer



      share|cite|improve this answer








      edited Aug 27 at 15:15

























      answered Aug 25 at 21:48









      mengdie1982

      3,643216




      3,643216











      • Please don't use JPG for non-photographic images.
        – Andreas Rejbrand
        Aug 26 at 9:41
















      • Please don't use JPG for non-photographic images.
        – Andreas Rejbrand
        Aug 26 at 9:41















      Please don't use JPG for non-photographic images.
      – Andreas Rejbrand
      Aug 26 at 9:41




      Please don't use JPG for non-photographic images.
      – Andreas Rejbrand
      Aug 26 at 9:41










      up vote
      2
      down vote













      Another Proof



      Denote the the antipodal point of $F$ as $J$. Draw a line $l$ passing through $J$, parallel to $BC$, and intersecting $AB, AC$ at $K, L$ respectively.



      Notice that $BCLK$ is a tangential quadrilateral. By Newton's Theorem(Page 156-157) ,which is a degenerated form of Brianchon's theorem, we may obtain $BL,CK,FJ$ are concurrent, namely at $H$. But $KL||BC$, hence $AH$ bisects $BC$ and $KL$. We are done.



      enter image description here






      share|cite|improve this answer


























        up vote
        2
        down vote













        Another Proof



        Denote the the antipodal point of $F$ as $J$. Draw a line $l$ passing through $J$, parallel to $BC$, and intersecting $AB, AC$ at $K, L$ respectively.



        Notice that $BCLK$ is a tangential quadrilateral. By Newton's Theorem(Page 156-157) ,which is a degenerated form of Brianchon's theorem, we may obtain $BL,CK,FJ$ are concurrent, namely at $H$. But $KL||BC$, hence $AH$ bisects $BC$ and $KL$. We are done.



        enter image description here






        share|cite|improve this answer
























          up vote
          2
          down vote










          up vote
          2
          down vote









          Another Proof



          Denote the the antipodal point of $F$ as $J$. Draw a line $l$ passing through $J$, parallel to $BC$, and intersecting $AB, AC$ at $K, L$ respectively.



          Notice that $BCLK$ is a tangential quadrilateral. By Newton's Theorem(Page 156-157) ,which is a degenerated form of Brianchon's theorem, we may obtain $BL,CK,FJ$ are concurrent, namely at $H$. But $KL||BC$, hence $AH$ bisects $BC$ and $KL$. We are done.



          enter image description here






          share|cite|improve this answer














          Another Proof



          Denote the the antipodal point of $F$ as $J$. Draw a line $l$ passing through $J$, parallel to $BC$, and intersecting $AB, AC$ at $K, L$ respectively.



          Notice that $BCLK$ is a tangential quadrilateral. By Newton's Theorem(Page 156-157) ,which is a degenerated form of Brianchon's theorem, we may obtain $BL,CK,FJ$ are concurrent, namely at $H$. But $KL||BC$, hence $AH$ bisects $BC$ and $KL$. We are done.



          enter image description here







          share|cite|improve this answer














          share|cite|improve this answer



          share|cite|improve this answer








          edited Aug 25 at 22:51

























          answered Aug 25 at 22:31









          mengdie1982

          3,643216




          3,643216




















              up vote
              1
              down vote













              $IH = JH$ because $M$ is the center point of BC and triangle $AIJ$ is similar with respect to the intersection of its parallel base with line $AM$.
              However, you have to show that these two sides are the same due to symmetry about the line $FD$, so the extension of $FD$ is shown to meet the intersection at $H$.



              The following diagram achieves this by adding a symmetrical construction about $FD$ with $X = Y$.



              enter image description here






              share|cite|improve this answer
























                up vote
                1
                down vote













                $IH = JH$ because $M$ is the center point of BC and triangle $AIJ$ is similar with respect to the intersection of its parallel base with line $AM$.
                However, you have to show that these two sides are the same due to symmetry about the line $FD$, so the extension of $FD$ is shown to meet the intersection at $H$.



                The following diagram achieves this by adding a symmetrical construction about $FD$ with $X = Y$.



                enter image description here






                share|cite|improve this answer






















                  up vote
                  1
                  down vote










                  up vote
                  1
                  down vote









                  $IH = JH$ because $M$ is the center point of BC and triangle $AIJ$ is similar with respect to the intersection of its parallel base with line $AM$.
                  However, you have to show that these two sides are the same due to symmetry about the line $FD$, so the extension of $FD$ is shown to meet the intersection at $H$.



                  The following diagram achieves this by adding a symmetrical construction about $FD$ with $X = Y$.



                  enter image description here






                  share|cite|improve this answer












                  $IH = JH$ because $M$ is the center point of BC and triangle $AIJ$ is similar with respect to the intersection of its parallel base with line $AM$.
                  However, you have to show that these two sides are the same due to symmetry about the line $FD$, so the extension of $FD$ is shown to meet the intersection at $H$.



                  The following diagram achieves this by adding a symmetrical construction about $FD$ with $X = Y$.



                  enter image description here







                  share|cite|improve this answer












                  share|cite|improve this answer



                  share|cite|improve this answer










                  answered Aug 26 at 0:23









                  Phil H

                  1,9472311




                  1,9472311



























                       

                      draft saved


                      draft discarded















































                       


                      draft saved


                      draft discarded














                      StackExchange.ready(
                      function ()
                      StackExchange.openid.initPostLogin('.new-post-login', 'https%3a%2f%2fmath.stackexchange.com%2fquestions%2f2894467%2fanother-beauty-hidden-in-a-simple-triangle-2%23new-answer', 'question_page');

                      );

                      Post as a guest













































































                      Comments

                      Popular posts from this blog

                      Long meetings (6-7 hours a day): Being “babysat” by supervisor

                      Is the Concept of Multiple Fantasy Races Scientifically Flawed? [closed]

                      Confectionery